• 0 Shopping Cart $ 0.00 -->

JD Advising

Did you fail the bar exam? We have lots of free resources to help you regroup for your next attempt! Check out our guide on what to do if you failed the bar exam , as well as our guide on hiring a bar exam tutor ! And be sure to register for our FREE Failed the Bar Exam Mastery Class (includes daily giveaways)!

Ace Your Law Finals! 🌟 Boost your GPA and class rank with expert private tutoring & premium Law School Study Aids . Elevate your legal mastery — Start Today!

Need a bar exam score boost? Looking for one-on-one attention in your bar prep? Our expert bar exam tutors are ready to help you pass! Sign up early before spots fill up!

Essays Are On The Bar Exam

Exactly How Many Essays Are On The Bar Exam?

The written component of the bar exam makes up a significant portion of your score.  In fact, in the vast majority of Uniform Bar Exam jurisdictions, the written portion makes up 50% of your final score.  But what does that mean in terms of what kind of writing you are expected to produce on the bar exam?  It’s not quite as simple as writing essay after essay for a full testing day.  In this post, we discuss how many essays are on the bar exam, as well as where the rest of your written score comes from.

How Many Essays are on the Bar Exam?

First, it is important to make sure you know exactly what kind of bar exam your jurisdiction administers.  Although nearly 40 states administer the Uniform Bar Exam now, and many more use the Multistate Essay Exam (MEE) as their essay component, there are still states that use their own format.  When wondering how many essays are on the bar exam, make sure you find the answer to this question for your specific bar exam.

For those in Uniform Bar Exam jurisdictions:

You will be taking the MEE as your essay component.  In these states, there are six essays on the bar exam.  Each state administers the same six essays.  These essays do not test state-specific law, just majority and minority approaches.  However, your state might also require a state-specific component in addition to the MEE, so keep that in mind.

In the vast majority of UBE jurisdictions, the MEE is worth 30% of your overall final score.  You will be expected to complete the six MEE essays in three hours, which means you should average about 30 minutes per essay.  You can learn more about how to approach the MEE, including formatting and studying tips, in this excellent free guide !

But that’s not all!  There are more than just essays on the bar exam, and on the written portion in particular.  UBE jurisdictions also administer the Multistate Performance Test, or the MPT.  This is worth 20% of your final score.  The MPT asks you to write two lawyerly tasks, such as a persuasive brief or objective memo, in three hours.  MPTs are different than essays because you are not expected to apply law that you call up from memory.  The MPT is a closed universe where you are given a set of facts and the law that this fake jurisdiction applies.  It is then up to you to draft a legal document that interprets the law and applies it to the facts to solve a problem or accomplish a goal.

Be sure to put in the time preparing for the MPT portion as well!  Each writing component is very important to your final score and requires different preparation.  With the MPT, practice makes perfect!  Try to practice at least one MPT for each of the different kinds of tasks you might encounter. Practice is important for the MEE essays on the bar exam as well, but you also need to make sure you have the law memorized!  When writing an essay on the bar exam, you will want to pick up as many points as possible by providing an accurate rule, a precise analysis, and a conclusion that follows naturally from that analysis.

By practicing your writing skills for both MEE essays and MPTs, as well as efficiently memorizing the law, you will set yourself up to do well on the written portion of the bar exam!

Looking to Pass the Bar Exam?

Free Resources:

  • 🌟 Bar Exam Free Resource Center : Access our most popular free guides, webinars, and resources to set you on the path to success.
  • Free Bar Exam Guides : Expert advice on the MBE, the MEE, passing strategies, and overcoming failure.
  • Free Webinars : Get insight from top bar exam experts to ace your preparation.

Paid Resources:

  • 🏆 One-Sheets : Our most popular product! Master the Bar Exam with these five-star rated essentials.
  • Bar Exam Outlines : Our comprehensive and condensed bar exam outlines present key information in an organized, easy-to-digest layout.
  • Exclusive Mastery Classes : Dive deep into highly tested areas of the MBE, MEE, MPT, and CA bar exams in these live, one-time events.
  • Specialized Private Tutoring : With years of experience under our belt, our experts provide personalized guidance to ensure you excel.
  • Bar Exam Courses : On Demand and Premium options tailored to your needs.
  • Bar Exam Crash Course + Mini Outlines : A great review of the topics you need to know!

🔥 NEW! Check out our Repeat Taker Bar Exam Course and our new premier Guarantee Pass Program !

Related posts

prepare for bar exam

Leave a Reply

Your email address will not be published. Required fields are marked *

Save my name, email, and website in this browser for the next time I comment.

  • Privacy Policy
  • Terms of Use
  • Public Interest

Scoring Highly on Multiple-Choice, Bad Interview Experience

By using this site, you allow the use of cookies, and you acknowledge that you have read and understand our Privacy Policy and Terms of Service .

Cookie and Privacy Settings

We may request cookies to be set on your device. We use cookies to let us know when you visit our websites, how you interact with us, to enrich your user experience, and to customize your relationship with our website.

Click on the different category headings to find out more. You can also change some of your preferences. Note that blocking some types of cookies may impact your experience on our websites and the services we are able to offer.

These cookies are strictly necessary to provide you with services available through our website and to use some of its features.

Because these cookies are strictly necessary to deliver the website, refusing them will have impact how our site functions. You always can block or delete cookies by changing your browser settings and force blocking all cookies on this website. But this will always prompt you to accept/refuse cookies when revisiting our site.

We fully respect if you want to refuse cookies but to avoid asking you again and again kindly allow us to store a cookie for that. You are free to opt out any time or opt in for other cookies to get a better experience. If you refuse cookies we will remove all set cookies in our domain.

We provide you with a list of stored cookies on your computer in our domain so you can check what we stored. Due to security reasons we are not able to show or modify cookies from other domains. You can check these in your browser security settings.

We also use different external services like Google Webfonts, Google Maps, and external Video providers. Since these providers may collect personal data like your IP address we allow you to block them here. Please be aware that this might heavily reduce the functionality and appearance of our site. Changes will take effect once you reload the page.

Google Webfont Settings:

Google Map Settings:

Google reCaptcha Settings:

Vimeo and Youtube video embeds:

You can read about our cookies and privacy settings in detail on our Privacy Policy Page.

National Conference of Bar Examiners

13 Best Practices for Grading Essays and Performance Tests

lightbulb with article title

This article originally appeared in The Bar Examiner print edition, Winter 2019-2020 (Vol. 88, No. 4), pp 8–14. By Sonja Olson

how many essays on the bar exam

When grading essays and performance tests for the bar examination, fairness, consistency, and focus are the cornerstones of good grading. In this article, NCBE’s MEE/MPT Program Director shares best practices for grading these written components to ensure that they serve as reliable and valid indicators of competence to practice law.

Opinions may vary about what should be tested on the bar exam, but if there is one point of agreement, it is that lawyers need to be skilled at communicating in writing. And communicating in writing means much more than using proper syntax, grammar, and vocabulary. Lawyers must be able to adjust their writing to a variety of audiences, such as clients, courts, opposing counsel, and legislators. Essay questions and performance tests are therefore integral to evaluating whether an individual should receive a license to practice law.

The Multistate Essay Examination (MEE) and the Multistate Performance Test (MPT) would not perform as reliable testing components without the dedication and care exercised by the graders in every jurisdiction that uses those exam components. In this article I share some insights and best practices that I’ve learned over the years from our Grading Workshop facilitators, from our MEE and MPT Drafting Committees, and without a doubt, from the graders (veterans and newcomers) who participate in NCBE’s Grading Workshop after every February and July bar exam administration. 1 Following these grading best practices ensures that the MEE and MPT serve as valid and reliable measures of basic competence to practice law.

1. Know the question (and the answer).

Every MEE question comes to the graders with the Drafting Committee’s analysis of the issues raised by the question and a discussion of the applicable law. In addition, we provide grading guidelines at the Grading Workshop. These guidelines, generally one to two pages, distill the issues discussed in the MEE analyses but also offer suggestions for distinguishing answers and may identify common areas where examinees struggle. This information is based on the workshop facilitator’s review of at least 30 actual MEE answers, which are sent to NCBE by jurisdictions after the bar exam. For the MPT, the drafters’ point sheet identifies the issues raised in the MPT and the intended analysis.

Familiarity with the grading materials not only allows a grader to give credit where it is due but also ensures that a grader can readily identify answers containing extraneous discussion that may be accurate (such as memorized portions of bar review outlines) but is not pertinent to a discussion of the issues raised by the problem.

Particularly with performance tests, which provide the relevant law, examinees may reiterate sentences or more from the statutes, regulations, or cases in the test booklet. MPTs also present a more expansive collection of facts for examinees to master, and thus there is the temptation to recite extended portions of the facts in an answer. Familiarity with the text of these questions, from the beginning of grading, will make it much easier to identify examinees who are merely regurgitating material as opposed to synthesizing the relevant facts and law and producing a cogent analysis.

2. Know the applicable law.

Graders of the MPT have the luxury of having all examinees working from the same legal authorities, as the MPT is a closed-universe exam—that is, all the relevant law is provided as part of the MPT. So an MPT grader should have no worries that an examinee is referencing an alternative, but valid, legal doctrine—if it’s not in the Library portion of the test booklet, it’s probably not analysis that should receive credit.

New MEE graders, however, often ask whether it is expected that answers to MEE questions will contain the same level of analysis and legal citations as provided in the Drafting Committee’s analyses. The short answer is no—the MEE analyses are very detailed because the MEE Drafting Committee recognizes that graders may be assigned to grade questions in subject areas that are not frequently encountered in their law practices. The MEE analyses contain the legal authorities relevant to the problem and often some background material to help orient the grader. Graders of MEE questions may want to review the authorities cited in the analyses or other treatises and casebooks if grading a subject outside of their regular practice area.

3. Know the grading scale that your jurisdiction uses.

At the Grading Workshop, NCBE uses a six-point scale when discussing the grading of essays and MPTs. Some jurisdictions use another scale, such as 1 through 7 or 1 through 10. What matters is that the score scale is manageable enough that graders can make consistent and meaningful distinctions among answers without getting frustrated by trying to determine where an answer fits on an overly granular scale. 2 All graders in a jurisdiction should be using the same scale and be in agreement on when an answer is so deficient as to warrant a zero ( see best practice #13 ).

Graders should also know whether their jurisdiction requires that the grades conform to a particular distribution, such as a curve or equal percentages in each grading category. Note that one method isn’t preferred over another—the point is that all graders should be on the same page. 3

4. Focus on rank-ordering.

No grader should bear the weight and added stress of believing that the grade they assign to an essay or performance test is what will tip the scale for that examinee and determine whether he or she passes the bar exam. The emphasis should be on rank-­ordering the papers, not on whether an individual paper receives a passing or a failing grade. The score given to an essay by a grader is essentially a “raw” score because those essay grades will be scaled to the jurisdiction’s Multistate Bar Examination (MBE) scores. 4 Only then will the “real” grade for that specific essay be determined, which will then be added to that examinee’s MBE score, other essay grades, and grades on any other bar exam components to produce the final score.

That being said, a grader should be able to articulate why a paper ends up at a different point on the grading scale vis-à-vis those papers receiving a higher or lower grade. At times, most papers may drop easily into particular “piles” on the grading scale based on simple criteria—for instance, that they cover the first two issues well but then do not reach a correct conclusion on the third issue.

Graders should turn off their inner editor and focus on how well the paper has answered the call and demonstrates the examinee’s ability to reason and analyze compared to the other papers in the pile.

5. Achieve calibration to ensure consistency in rank-ordering.

Fairness to all examinees means that it shouldn’t matter when their papers are graded or by whom. Calibration is the means by which graders develop coherent grading judgments so that rank-ordering is consistent by a single grader as well as across multiple graders. The recommended practice is that a grader review at least 30 papers before grading “for real” to see what the range of answers is. Note that for both multiple graders and single graders, answers for each point on the grading scale should be identified before the “real” grading begins. This could require reviewing more than 30 papers.

For multiple graders: Reviewing at least 30 papers works well when there are two or more graders for a question. As graders read the same papers in the calibration packet, they should pause after every five or so answers and discuss what grades they have assigned. If they have graded the same papers differently, they should discuss those papers and come to an agreement for each paper. This process of grading, discussing, and resolving differences should continue through the whole calibration packet or until graders are confident that they are using the same criteria to differentiate papers.

For a single grader: For a single grader, it is just as important to review a calibration packet of 30 or more papers. The papers can be sorted into piles for each point on the grading scale. After reviewing the first 10 or 15 papers, the grader should revisit the grades given to the first papers to see if the initial grade still holds or if the paper in fact belongs in a different pile. Each pile should then be reviewed to verify that the papers in it are of a consistent quality. One approach that some graders have found helpful is to first separate answers into three piles (poor, medium/average, and good) and then review the papers in each pile, separating them into the 1s and 2s, the 3s and 4s, and the 5s and 6s.

6. Combat “grader drift.”

Graders can “drift,” or begin grading papers inconsistently, for a variety of reasons. Fatigue is a common reason, as is hitting a string of very poor (or very good) papers so that the next one seems very good (or very bad) when it is merely average. To ward against “grader drift,” all graders should have some answers from the calibration packet embedded into the papers they grade, with the score from the calibration session hidden. After grading the embedded paper (which may be on colored paper or otherwise marked as a part of the calibration set), the grader can compare the just-­assigned grade with that from the calibration session and determine whether drift is occurring. For multiple graders, embedding the same answer from the calibration packet at the same point in each grader’s pile provides an opportunity to check that the graders are internally consistent and still applying the same standards.

7. Spread out grades over the entire score scale.

Rank-order grading only works as an effective assessment tool if graders take care to use the entire score scale. This does not mean that the final grades fit a particular curve. Rather, even a grader in a smaller jurisdiction who has fewer than 100 papers to grade should have no problem finding papers that slot into each point on the scale. There will be fewer 1s and 6s (in the case of a 6-point scale) and likely more 3s and 4s, with the number of 2s and 5s probably falling somewhere in between—but there will be papers that a grader, with confidence and justification, may reasonably place at each given point on the scale. While there are times when a question may be easier and most examinees appear to do well, a grader will still be able to find valid points of distinction among answers that will allow the grader to spread out the scores. 5 Some graders may find it helpful to initially use pluses and minuses when grading and then to review those 4- and 4+ answers, for example, to see if they really belong in the 3 or 5 piles.

The table below illustrates the importance of spreading out grades. If there are two graders, each grading answers for different questions, and Grader A decides to use the whole score scale of 1 through 6 but Grader B thinks that all examinees performed about the same and gives out only 3s and 4s, the resulting combination of the scores from Grader A and Grader B demonstrates that it is really Grader A who, by taking care to use the entire range of possible scores, is determining how well, or how poorly, each examinee does overall.

Lack of calibration between graders of the same question is unfair to examinees because their scores will be affected not by the quality of their answers but by whether they got the “easy” or the “hard” grader. On a similar note, if graders of different questions fail to spread out their grades, the questions whose grades are “bunched up” will ultimately have less impact on examinees’ overall scores.

8. Approach each paper as an “empty bucket”—that is, look for reasons to give credit.

Just as we encourage graders at the Grading Workshop to avoid thinking that the pass/fail line is whether a paper receives a 3 as opposed to a 4 on a 6-point scale, we encourage graders to approach each paper as an “empty bucket” and to view their task as searching for points to add to the bucket. It is much more likely that a grader can be consistent across papers in what he or she will give credit for, instead of attempting to be fair and consistent in all the ways a paper could be penalized.

9. Grade in a compressed time period.

Some jurisdictions, where the number of examinees means that grading cannot be completed over the course of a long weekend, may set targets for the number of papers that a grader can reasonably grade in a day. Certainly, grading is not something that should be rushed. But it is much easier to maintain calibration if the grader doesn’t have to get reacquainted with the details of the legal analysis and the quality of the answers because of a start-and-stop grading process spread out over several weeks. To the extent possible, grading should be done over a shorter time period.

10. Know the additional factors to consider when assigning grades.

Know what factors are legitimate grounds for assigning different grades to papers. Obviously, the content and substance of the answer is the first indication—what parts of the question did the examinee answer correctly? But other qualities are valid reasons for distinguishing papers.

Response to the call of the question

For both MEEs and MPTs, the answer should respond to the call of the question asked—and not the question that the examinee may have preferred to answer. For example, the examinee may launch into a discussion of whether a contract was validly entered into when the call of the question specifically asks for an assessment of the amount of damages the plaintiff is likely to recover. If that examinee then goes on to provide discussion that does respond to the specific call, the examinee will receive credit for the good content and at that point, the grader can generally ignore the extraneous material. Examinees who inflate their answers with a lot of extraneous material effectively penalize themselves: including the irrelevant material leaves the examinee less time to devote to the legal issues that are raised by the call. If two papers have approximately equal good content, but one is cluttered by unnecessary material, the one that adheres to the relevant issues is the better answer, although depending on the overall group of answers, those two papers could end up in the same pile.

Additional factors to consider, especially with MPTs, are the answer’s format, structure, and tone, and whether the examinee followed directions (e.g., if the task is to draft a letter to opposing counsel, which should be a persuasive piece, and instead the examinee writes an objective memorandum, this should be taken into account in determining the examinee’s grade). Finally, the analysis should state the applicable legal standard, marshal the relevant facts, and apply the law to those facts in the problem.

Accuracy in stating facts

In a similar vein, examinees may get a fact or two wrong when writing their answers. Even with MEE questions, which are generally just one page long, in the rush to produce an answer in 30 minutes, it is not unusual for an examinee to misread or misstate facts. The mistake may be very minor (e.g., getting a character’s name wrong). If it is clear from the context whom the examinee is discussing, such an error can probably be ignored. But if an examinee misstates a fact and then hinges part of the analysis on that incorrect statement, that should likely be considered when grading. After all, an important lawyering skill is paying attention to the facts that matter and getting them right when presenting a legal argument or analysis.

Written communication skills

For jurisdictions that have adopted the Uniform Bar Examination (UBE), the UBE Conditions of Use mandate that graders take into account written communication skills when grading, although no discrete weight is provided for that component. Each jurisdiction may have specific guidelines for how its graders should handle papers that are riddled with typos, exhibit poor grammar, or contain irrelevant information (legal or factual), among other things. Obviously, there will be a point where a paper’s typos and poor grammar will make it impossible to discern whether the examinee does comprehend the relevant legal principles, and in such cases, a lower grade is warranted. But typos and occasional poor grammar, in themselves, should generally not factor into the grading decisions for most papers. NCBE suggests ignoring typos for the most part because it is unreasonable to expect perfection in typing skills given the time pressure of the exam.

When assessing the quality of the writing, the focus should generally be on characteristics such as logical and effective organization, appropriate word choice and level of detail, and the presence or absence of a clear conclusion. The quality of the writing does matter, and while it remains important in MEE answers (and its absence is more obvious, if only because MEE answers are fairly brief), it comes to the forefront when grading MPTs. For one thing, while essay prompts ask the examinee to provide solid, reasoned legal analysis, the MPT instructs the examinee to consider both the audience of the work product and what tone is called for, objective or persuasive, to properly complete the task.

11. Know when to assign partial credit.

Essay exams are more forgiving than multiple-choice questions. If an examinee taking the MBE knows the relevant legal rule and is able to narrow the answer down to two options, one of which is the correct answer, but still selects the wrong answer, the examinee receives no credit for that question. The Scantron machine doesn’t care how close the examinee came to the right answer. But essay questions give examinees a chance to earn partial credit—they have an opportunity to demonstrate their ability to identify relevant facts and employ legal reasoning to reach a conclusion. Even if the ultimate conclusion is incorrect, an examinee who has stated the correct legal rule and then produced a cogent analysis of how the law would apply should get substantial credit. Graders should spend enough time on each paper to see where the examinee has shown some knowledge of the law and how it would apply to the given situation, even if the examinee does not reach the “correct” conclusion.

Similarly, just because an examinee hasn’t remembered the correct name of a legal doctrine, that doesn’t exclude that paper from receiving at least some credit. Depending on the range of quality of answers, an examinee should receive some amount of credit, even substantial credit, for describing the applicable rule or doctrine. The grader should ask whether the examinee’s discussion indicates that he or she is applying the same criteria covered by the relevant doctrine.

12. Acknowledge when a paper is incomplete.

With incomplete papers, those where the examinee clearly ran out of time (sometimes as obvious as a final sentence that cuts off, or a missing final issue, or analysis that starts strong but gets more superficial and conclusory toward the end), the grader can’t provide the answer that the examinee didn’t get to, no matter how promising the first paragraphs are. Fairness to all examinees requires that a grader award credit only for what is on the page, as other examinees were able to complete the essay or performance test in the time allowed by appropriately managing their time.

13. Know when to assign a zero.

All graders in a jurisdiction should be in agreement about when a paper should receive no credit, that is, a zero. A score of zero should be reserved for a blank page or an answer that is completely nonresponsive to the call of the question. This is important because essay answers that receive a zero are excluded from the reference group that is used to determine the formula for scaling essay scores to the MBE. Earning a 1 instead of a 0 should require that the examinee has made an honest attempt to answer the question.

Fairness, consistency, and focus are the cornerstones of good grading. Following these practices in grading bar exam essays and performance tests will not lessen the workload, but it will help ensure that bar exam essays and performance tests serve as reliable and valid indicators of an examinee’s competence to practice law, that scores are fair to examinees and are the result of meaningful differences in the quality of the answers, and that the quality of the writing—an important skill for all lawyers, regardless of practice area—is considered as a grading criterion.

  • NCBE’s MEE/MPT Grading Workshop is held in Madison, Wisconsin, the Saturday after each administration of the bar exam. The purpose of the workshop is to identify trends that graders will likely see when grading the MEE and the MPT in their jurisdictions as well as to discuss any questions graders have about the applicable law or the grading materials. While the workshop gives graders an orientation for grading, it is not intended to be a calibration session; that is best accomplished using a calibration packet comprising papers solely from the grader’s jurisdiction. (Calibration is the means by which graders develop coherent grading judgments so that rank-ordering is consistent by a single grader as well as across multiple graders.) (Go back)
  • See Mark A. Albanese, PhD, “ The Testing Column: Essay and MPT Grading: Does Spread Really Matter? ” 85(4) The Bar Examiner (December 2016) 29–35, at 30: “For the purposes of illustrating how spread in grades affects the [standard deviation—that is, the average deviation of scores from the mean—] a six-point scale works fairly well. There are enough different grade points that spread can be easily seen, yet not so many that one gets lost in the details of computation.” See also Susan Case, PhD, “ The Testing Column: Bar Examining and Reliability ,” 72(1) The Bar Examiner (February 2003) 23–26, at 24: “All else being equal, more score gradations work better than fewer score gradations. The key is to make sure that the scale reflects the level of judgments the grader can make…. A six-point grading scale tends to work better than a four-point grading scale. Something much broader, like a 20-point grading scale, would work better than a six-point scale, but only if the grader could make reasonable, consistent, meaningful decisions along that scale.” (Go back)
  • See Mark A. Albanese, PhD, supra note 2, at 32: “From a practical standpoint, we want to spread scores out as much as possible, but it is not necessary for the number of essays to be evenly distributed in each grade category; there are a range of distributions that achieve reasonably spread-out grades, but they tend to involve having some percentage of examinees in each grade category and not “bunching up” examinees too much into a small number of grade categories. In other words, uniform and bell-shaped distributions of grades are reasonable ways of “bucketing” examinees to ensure good spread in grades.” (Go back)
  • Scaling is a procedure that statistically adjusts raw scores for the written components of the bar exam (the MEE and the MPT) so that collectively they have the same mean and standard deviation (average distance of scores from the mean) as the jurisdiction’s scaled MBE scores. See Susan Case, PhD, “ The Testing Column: Frequently Asked Questions about Scaling Written Test Scores to the MBE ,” 75(4) The Bar Examiner (November 2006) 42–44 at 42: “In the bar examination setting, scaling is a statistical procedure that puts essay or performance test scores on the same score scale as the Multistate Bar Examination. Despite the change in scale, the rank ordering of individuals remains the same as it was on the original scale.” (Go back)
  • Scaling ( see supra note 4) takes advantage of the equated MBE scores and therefore accounts for variance in difficulty of the essay questions from one administration to the next. See Mark A. Albanese, PhD, “ The Testing Column: Scaling: It’s Not Just for Fish or Mountains ,” 83(4) The Bar Examiner (December 2014) 50–56, at 55: “Scaling essay scores to the MBE will … stabilize passing rates even though the intrinsic difficulty of essay questions may vary.” (Go back)

Portrait photo of Sonja Olson

Sonja Olson is the MEE/MPT Program Director for the National Conference of Bar Examiners.

Contact us to request a pdf file of the original article as it appeared in the print edition.

In This Issue

Winter 2023-2024 (Vol. 92, No. 4)

how many essays on the bar exam

  • Letter from the Chair
  • President’s Page
  • Facts & Figures
  • Military Spouse Attorney Licensure: Progress and Perspectives
  • Distance-Education Allowances and Academic Freedom in Law Schools: Recent Developments in the ABA Standards
  • Texas Bar Exam: 2023 Standard-Setting Study
  • The Testing Column: Standards? We Don’t Need No Stinking Standards!
  • The Next Generation of the Bar Exam: Quarterly Update
  • FAQs About Bar Admissions: Answering Questions About: Transferring UBE Scores Between Jurisdictions
  • News & Events
  • In Memoriam: Mark A. Albanese, PhD
  • In the Courts

Bar Exam Fundamentals

Addressing questions from conversations NCBE has had with legal educators about the bar exam.

Online Bar Admission Guide

Comprehensive information on bar admission requirements in all US jurisdictions.

NextGen Bar Exam of the Future

Visit the NextGen Bar Exam website for the latest news about the bar exam of the future.

BarNow Study Aids

NCBE offers high-quality, affordable study aids in a mobile-friendly eLearning platform.

2023 Year in Review

NCBE’s annual publication highlights the work of volunteers and staff in fulfilling its mission.

2022 Statistics

Bar examination and admission statistics by jurisdiction, and national data for the MBE and MPRE.

Make This Your Last Time - A Candid, No-BS Look at Bar Exam Preparation

Make This Your Last Time

Bar Exam Preparation

Dominating the Essays: Organize Issues and Prioritize Rules to Know on the Bar Exam

Ever wonder how you’re supposed to juggle everything in your head? How do you prioritize the rules to know for the bar exam?

How are you supposed to learn all this when time is tight? How do you tackle the massive body of rules to know?

How do you know you’ve completed the essay in full? Did you even talk about the correct issues? Are the graders going to give you the points? Are they even going to read your prose?!

You’d love to start practicing essays but feel like you just haven’t learned enough law yet. It’s overwhelming to even begin.

At least the answer is right there in MBE questions… If you’re a bar taker struggling with coming up with what to write, essays are the bane of your existence. Your rambling paragraphs start to blur.

Let’s breathe. We can simplify the essays and make them less scary…

Key takeaways:

  • Issues: Learn not just the rules but also how to present and organize the issues (with examples below)
  • Rules: Highest-priority issues and rules are those that have appeared in the past (there are two other priorities)
  • There are efficient and effective ways to hit both of the above at once

Know not just the rules but also the issues

Put yourself in the essay grader’s shoes for a moment…, so now, as the applicant, what can you do to get the most points on the essays, 1) don’t write like a lawyer. write like a bar taker., 2) organize the issues., prioritize memorizing the rules and issues in this order…, 1) issues and rules that have appeared in past exams, including main issues, sub-issues, defenses, 2) issues and rules deemed important, 3) other fringe issues and rules that might come up (rule against perpetuities, anyone), take your practicing and memorizing to the next level.

Before getting into how to prioritize the rules for the bar exam, I need to emphasize something else first.

After working with readers and coaching clients over the years, there are two things that have become apparent:

  • Writing essays on the bar exam is NOT about writing like a lawyer
  • Issues are king (I talk about this more in this video )

That’s why gaining experience and intuition through writing bar essays is so helpful. You won’t be too surprised or lost on what to write about during the exam.

Meanwhile, some bar takers think they have to write a beautiful treatise so that bar graders can put on a monocle, do a deep literary analysis of the romance between P and D, and press it against their chest while looking out the window because your prose tugged at their heart.

The graders couldn’t care less about that LOL

If it helps, think about the time you watched someone slowly lose interest in you while being unable to do anything about it. And how futile texting them your thoughtful walls of text was.

You sigh at the pile of essay answers that waits for you every day.

You have thousands of the same shitty ass essays to sift through. You have to try to be consistent and fair across all those essays. You try your best at your thankless job, and all they do is complain about your “subjectivity.”

You also want to get through these essays as quickly as possible since you’re already getting underpaid. You’re already reading these answers while at the red light or sitting on the toilet. Your life is blurring together.

Perhaps it’s time to cut corners. Maybe just look for the headings and scan for keywords in the analysis…

This becomes a fairly mechanical process once you get through several similar issue patterns. Like fact patterns , there are corresponding issue patterns .

Prior law experience or creative writing will detract from answering the way graders want you to.

Practicing attorneys tend to not do as well on bar essays because this is a SEPARATE skill from real practice.

How do you write the way the graders want to see? Write like a bar taker instead:

Make the issues loud and clear. Punch the grader in the eyes by creating clear headings. Break out the sub-issues and elements into their own “street signs” for the grader.

Here’s an example of a major issue ( Contract Formation ) and an element (Offer) clearly called out, and their corresponding principles (rule statements):

How to organize issues for bar essays

In fact, being able to identify (or “spot”) and organize the issues is at least as important as knowing the rules. ( Here’s how to make issue identification a systematic process. )

It’s not about long-winded analyses or writing beautiful rule statements (neither got me to pass the first time).

Issues are where everything starts. An IRAC can’t sprout (and you get no points) from a seed that’s never planted.

Identifying the relevant issues is a signal to the grader that you understand what’s being tested. Rules and applications will naturally cascade down from the issues like a waterfall. Yes, you still want the meat of your answer written well.

Think of it as submitting a resume (or texting a girl). Hit the main points that the reader will care about. You’re but one out of a pile. Just a number. A hassle to go through. Better make the most out of the 10-15 seconds of the recruiter’s attention.

There is a certain logic and order to how you approach each issue.

Example of how to organize a call of the question in an Evidence essay:

Issue outline - Evidence

You can even start to notice common issues that clump together (that you should discuss to get as many points as possible):

Common issue clumps

Here’s one for a Civil Procedure essay. Just plug and play the rules:

Issue outline - Civil Procedure 2004 Feb

This is like 75% of the essay, so don’t freak out if it looks like a lot. But it’s a lot more structured than trying to start by typing a mess of words, right?

The best part: These issues clumps repeat! There are issue patterns like I said above. So if you see another SMJ or PJ question, just plug and play the same thing as here.

Outlining the issues and filling in the blanks should become a routine process because you will have seen the patterns so many times. Success is boring, not sexy.

If you have issues outlined like this, you’re pretty much home free. Do this for every essay, and you’ll know how to solve similar essays that appear on the bar exam.

OK, so that was about issues. Now about prioritizing rules…

Of course, you still need to know the law corresponding to the issues you identify. But if you need to prioritize, narrow the field of issues and rules in this order:

Issues and rules that have been tested a lot tend to be tested again. The more they’ve been tested, the more important they are.

Learn the important issues and rules by solving problems from the past and studying the answers.

  • Find a collection of past essays and PTs here (for the California and Uniform Bar Exams).
  • Find real MBE questions from sources such as Emanuel’s Strategies & Tactics for the MBE or AdaptiBar .
  • Here are the biggest (highly tested) areas on the MBE.

You don’t need to have “learned enough law yet” before you dive into the pool! Trying to ensure that is exactly how I failed the California Bar Exam my first time.

Knowledge removed from the facts is nothing. It’s all artificial if you don’t know how to use it.

Seeing real problems will be productive because it will reveal what you know and don’t know. It’s like getting on the scale to measure yourself. THEN you can review and fill in the gaps.

The essay and MBE questions you go through now will become familiar fact patterns you might see again on the exam. Practicing and self-critiquing your work help you accomplish everything you seek:

  • Getting better at identifying issues
  • Memorizing and remembering rules through active recall
  • Knowing how to apply the rules you memorized
  • Picking the right answer on the MBE more often
  • Gaining confidence

In other words, practicing will help solidify everything, including understanding and retaining the important concepts likely to be tested. Exciting!

You may not get through all the past essays, but you may have a feeling that there are issues and rules that would be good to know.

Maybe you’ve seen them around somewhere, maybe in law school or mentioned in a bar prep lecture. Typically, they’re listed in outlines .

You’ll still need to rote memorize these things, unfortunately. MBE questions especially may test you on specific obscure rules.

Note that I continue to mention issues and not just rules. That’s because memorization isn’t just about memorizing rules .

Better to at least get familiar in case they ask you about it.

When I retook the bar exam, there were essay questions I wasn’t sure how to answer because I didn’t know about zoning (issues I didn’t know about) and criminal prosecutor ethical duties (rules I didn’t know).

Even though my life flashed before my eyes thanks to these gaps in knowledge, I still passed the exam by focusing more on the first two priorities during bar prep than absolutely everything at once .

Now you know where to focus if you’re short on time. If you feel tempted to skip over subjects based on predictions , try this approach instead.

It’s still a lot to learn, though.

If you want to make the material more manageable and less overwhelming, check out Magicsheets and Approsheets.

Magicsheets condensed outlines contain all three of the above categories of priority—covering 95% of the testable issues and rules in 5% of the space of your bar prep course content.

Approsheets issue checklists and flowcharts help you hit all the relevant issues on an essay so you can stop having that “blank screen syndrome.”

It’s a no-brainer if you want to invest in your dreams:

"Your blog, Magicsheets, and Approsheets were the deciding factor in my passing the California bar last year after ditching Barbri.  I am now enjoying my job in Big Law in CA, and can't speak highly enough of you and your product to my friends. ... Your material is much more helpful than any treatise or guide."

  • Changes to the California Bar Exam in 2017 July (Format, Grading, and How to Prep)

One Reply to “Dominating the Essays: Organize Issues and Prioritize Rules to Know on the Bar Exam”

I paid for my materials and I have not received them.

Leave a Reply Cancel reply

Your email address will not be published. Required fields are marked *

This site uses Akismet to reduce spam. Learn how your comment data is processed .

[email protected]

Critical Pass

How You Can Master Essays on the Bar Exam

06 Dec, 2021

The 3 Components of the Bar Exam

The bar exam consists of three main components in most jurisdictions: multiple choice, essays, and a performance test.

Excelling on each requires different approaches . The multiple choice section (called the "Multistate Bar Exam") is the toughest and counts for a significant part of your total score . The performance test ("PT") is the easiest is the least important to your total score. The PT assesses skills more than knowledge. There's not a lot of studying you need to do other than taking a couple of practice PTs.

The essays count for a lot of your total score, and while they can appear difficult, they're not if you know what you're doing. And knowing what you're doing doesn't necessarily mean knowing the legal issue being tested in and out. Rather, it means understanding how to write an essay so it looks like you know the law and how to apply it.

How essays are scored & how they can be gamed.

To understand why essays are an art, it's important to think about how the bar exam is scored. On the multiple choice MBE, a machine scans your answers and determines how many you got right and wrong. In other words, the MBE is binary — you either know the answer or you don't. There's no middle ground.

Essays, on the other hand, are graded by humans. These essay graders have a finite amount of time to read and grade hundreds (if not thousands) of essays on the same question. It's not an enviable task. 

We'll let you in on a secret — bar exam essay graders are not reading each answer carefully. They are reading quickly to determine how well you, the bar examinee, understand the question and how to analyze the answer. In doing so, they're looking for key terms that a proper analysis of the question would require.

Additionally, essay graders are scoring your answers on a point scale, usually 1-5. So, unlike the MBE, even if you don't know part of an answer, you can still get points for what you do know. This is huge. And it's why you should never give in or feel defeated if you're stumped by an essay.

Essays test big issues more than wrinkles.

While the MBE always includes questions on minor exceptions or wrinkles, essays are different. Essays almost always test major, large legal issues. To the extent they test small legal exceptions or wrinkles, it comes as a small part of the question.

This means that for every main issue on the bar exam, you should understand the proper analysis, terms, elements, and tests involved. For example, for Equal Protection, know the standards of review and when they apply. For a Contracts question, make sure you're referencing key issues like offer, acceptance, and consideration.

How to game a bar exam essay.

The most important thing you can do in creating a good bar exam answer is write a clear, organized answer that discusses key facts and legal terms involved in the essay question.

Briefly outlining your answer before you start writing is particularly important to ensuring you do this well. Use either the IRAC method (issue, rule, analysis, conclusion) or the CRAC method (conclusion, rule, analysis, conclusion) — whichever you're comfortable with is fine.

Make sure you're writing concise paragraphs and sentences . Why? It makes reading your answer quickly much easier. An essay answer filled with half-page paragraphs and run-on sentences will not help you.

The takeaway.

On bar exam essays, showing you know what issue is being tested is as important as how well you know the issue. If you know the issue, you know the key terms, elements, standards, etc. involved, even if you don't know the answer. That alone is enough to create a really good bar exam essay. The key is writing clearly and concisely, making sure you're including the key terms, elements, etc. involved, and referencing the facts at hand.

YOU CAN PASS THE BAR EXAM .   CRITICAL PASS CAN GET YOU THERE.

*  MBE  and  MEE Essay Flashcards  that help you learn and memorize key bar exam concepts, effectively and affordably.

MEE Essay and MPT Flashcards

MBE Flashcards

MBE Flashcards

Critical Pass App

Critical Pass App

Age verification.

By clicking enter you are verifying that you are old enough to consume alcohol.

Shopping Cart

Your cart is currently empty..

  • Skip to primary navigation
  • Skip to main content
  • Skip to primary sidebar
  • Skip to footer

Bar Exam Toolbox®

Get the tools you need for bar exam success

How to Practice Essays in the Final Weeks Before the Bar Exam

February 5, 2020 By Natalie Nicol Leave a Comment

How to Practice Essays in the Final Weeks Before the Bar Exam

In order to successfully prepare for and pass the bar exam, you must have a strategy for practicing essay questions. It’s especially important to maximize your efforts on practice essays in the final weeks before the bar exam. This post outlines some tactics you can implement to make the most out of the practice essays you do in the three weeks leading up to the bar exam.

Do a Self-Assessment and Make a Practice Plan

You’ve made it to the final three weeks before the exam. At this point, you should be done with learning the substantive law and should be focused primarily on review, rule memorization , and timed practice. This is a great time to stop and do a self-assessment so that you can use your remaining time effectively.

Take a look at all of the subjects tested on the essay portion of the exam in your state. Be honest with yourself about how you feel about each of the essay topics and how much time you’ve dedicated to memorization and practice in each. Rank the subjects from “most confident” to “least confident,” and make a plan to devote extra time to your weaker subject areas.

Although it’s tempting to keep reviewing the rules and subjects you know best, now is the time to focus on your areas for improvement. You must expose yourself to the different ways issues in these subjects are tested on the essays and reinforce your understanding of the law. You should do at least one extra timed practice essay for all of your weaker subject areas (ideally, you’ll do two or more). For your stronger areas, you can continue reviewing your memorization outlines and may want to simply review fact patterns and outline essays on scratch paper in order to stay fresh.  

Transition to Closed-Book Practice

If you haven’t already, you must transition to closed-book practice no later than three weeks before the bar exam. Many students think that they can’t start doing closed-book practice until they’re “ready.” The reality is that it’s much better to realize that you don’t know a certain rule or how to analyze an issue from the comfort of your own home than in the testing center on exam day. You must push yourself to get out of your comfort zone and start doing closed-book practice. This is the best way to prepare for exam day and will help you feel confident when you open the essay packet.   

Critically Review Sample Answers

In the weeks leading up to the exam, you may feel like you’re in a time crunch and that doing as many practice essays as possible is the best use of your time. However, simply completing a practice essay without critically evaluating your performance will not help you prepare or improve.

After every timed practice essay, set aside at least 30 minutes to review your answer and compare it to sample answers from past bar exams. In most states, samples released by the state bar typically go far beyond what is needed to achieve a passing score. Nevertheless, you’ll want to check your practice essay against these answers and make sure that you have spotted all of the major issues and that your rule statements are precise. If you’re in California, you may want to purchase a subscription to BarEssays.com , which has a database of real scored essays for you to use as grading rubrics.

If you’ve missed issues, go back to the fact pattern and make sure that you understand which facts were relevant to this issue. If your rule statements are lacking, revisit your memorization outline and make sure that your materials are accurate. Commit the correct rule to memory and move on to the next practice essay.

Issue Spot and Outline Essays

The experienced bar exam tutors at Bar Exam Toolbox advocate for doing as much timed practice as possible . We also understand that you’re human and only have 24 hours in a day. If you need to take a short break from timed full practice essays or have only a few extra minutes to spare, you can still benefit from issue spotting and doing a scratch-paper outline for an essay. There are only so many different ways that bar examiners can test issues on essays. Reviewing a fact pattern and doing an outline can help you hone your issue-spotting skills and refine your strategy for outlining and preparing to write your essays.  

Use Essay Predictions Wisely

In many states, bar prep companies or local attorneys will release essay subject predictions. It can be very tempting to focus only on the subjects that these purported “gurus” forecast for the exam. Doing so is a risk that has come back to haunt many bar applicants when these predictions turn out to be wrong. If you rely on predictions at all, consider doing one or two extra practice essays in the subjects that seem more likely to be tested than others. You must be prepared to be tested in all of the possible subjects.

Make the Most of Your Remaining Time and Prepare to Pass!

With a lot of hard work and dedication, you can make a significant amount of progress on the essay portion of the bar exam in a short period of time. By implementing these strategies, you can study smart (not just hard) and make the most of your remaining time before the bar exam. Good luck!

how many essays on the bar exam

Ready to pass the bar exam? Get the support and accountability you need with personalized one-on-one bar exam tutoring or one of our economical courses and workshops . We’re here to help!

' src=

About Natalie Nicol

Natalie Nicol is an in-house corporate lawyer who works on a broad range of legal issues, including intellectual property, privacy, employment, corporate, transactional, and tax matters. She is an Internet Law & Policy Foundry Fellow and Project Manager for Without My Consent, a non-profit focused on combatting serious online invasions of privacy. Natalie earned her J.D. from UC Hastings College of the Law in 2013 with a concentration in Intellectual Property Law and her B.A. from Arizona State University in 2008. During law school, she was an intern at the Berkman Center for Internet & Society at Harvard University (now the Berkman-Klein Center), the Electronic Frontier Foundation (EFF), and the First Amendment Project. In her spare time, she enjoys yoga, cooking, and live music.

Reader Interactions

Leave a reply cancel reply.

Your email address will not be published. Required fields are marked *

Save my name, email, and website in this browser for the next time I comment.

Need to Pass the Bar Exam?

Sign up for our free weekly email with useful tips!

  • Terms & Conditions

Copyright 2024 Bar Exam Toolbox®™

BarMax Logo

Demystifying the Bar: What Exactly Is On the Bar Exam?

By Celeste Boles Mehran Ebadolahi -->

Light bulb legal scales bar exam

After years of education and hard work, you are finally ready to clear the last hurdle to your legal career: the Bar Exam! Before you dive into bar study, make sure you have a clear understanding of the schedule and format of the bar exam, and the subjects tested . After all, your entire study plan should be crafted to help you pass the test as efficiently as painlessly as possible – a task that’s hard to focus on without understanding the exam itself.

Bar Exam Schedule and Components

Generally, the bar exam is a two-day exam offered twice a year: the end of February and the end of July. There are some exceptions. For example, Louisiana and Nevada employ a 3-day test format. And, in many jurisdictions, students who receive testing accommodations may be permitted to spread their exam over additional days of testing. (For advice on securing accommodations, check out our blog post and free video on that topic!)

TUESDAY: Writing Essays and Performance Tests

The first day of the bar exam consists of essay questions and Performance Tests. Currently, paper materials containing the questions are provided to the examinees, who then type their answers into test-taking software on personal laptops which they bring to this day of the exam. (There is also a handwriting option, but we do not recommend handwriting unless you have a disability that makes using a computer unfeasible for you.)

Essays: If you took any closed-book “issue-spotter” essay type questions in law school, the essay component of the bar exam may look somewhat familiar! Bar Exam questions test your knowledge of the law and your skills in legal analysis: recognizing applicable legal issues, stating the rules that apply, applying those rules to the facts, considering counterarguments, and reaching well-reasoned conclusions. Incorporating plenty of real bar exam essay questions into your study will enable you to master the essay-testing skills specific to your bar exam and will bolster your understanding of bar-tested law.

Performance Tests: Performance Tests may be less familiar to most examinees. According to the NCBE, Performance Tests are meant “to test an examinee’s ability to use fundamental lawyering skills in a realistic situation and to complete a task that a beginning lawyer should be able to accomplish.” This part of the bar exam does not test substantive knowledge, so, while a background understanding of how the legal system works is fundamental, there is no specific law to memorize.

However, that does not necessarily mean that the Performance Test is easy! The Performance Test requires examinees to read and analyze a large amount of material: a Task Memo setting forth the assignment; a Library of statutes, cases, and other sources of law; and a File consisting of notes, documents, and transcripts containing the relevant facts of the case. But reading is just the start – you’ll also need to prepare a complete and coherent legal document, with a reasoned legal analysis that considers all of the provided law and precedents. That can be daunting in 90 minutes! For this reason, timed practice is crucial , so that you can master Performance Test writing skills and become comfortable with a method of organizing the information and a pacing strategy that works for you.

What Does The Day Look Like In Your State?

UBE: The first day of the Uniform Bar Exam (administered all of the US jurisdictions listed here) consists of 6 Multistate Essay Exam (MEE) essay questions of 30 minutes each , followed by two Multistate Performance Tests (MPT) of 90 minutes each. Some states switch the order: Performance Tests in the morning, essays in the afternoon. Check with your jurisdiction to be sure! Both the MEE and MPT are drafted by the National Conference of Bar Examiners (NCBE) and are identical in all UBE states.

California: The first day of the California Bar Exam consists of 5 essays questions of one hour each , followed by one California Performance Test. There is a break for lunch between the third and fourth essay questions.

Other States:

  • Hawaii’s bar exam is identical to the UBE, with the addition of 15 multiple choice questions constituting the Hawaii Legal Ethics Exam.
  • South Dakota uses 5 essays from the MEE and one essay on Federal Indian Law, plus two Performance Tests.
  • Wisconsin can administer any combination of MEE, MPT, and locally drafted questions.
  • The writing portion of the Georgia and Delaware exams use the MPT alongside state-specific bar exam essay material.
  • Nevada administers the Nevada Performance Test, which is identical in format to the MPT and the California Performance Test but locally drafted, together with Nevada state-specific bar exam essays.
  • Florida, Mississippi, Nevada, and Virginia use only state-specific materials for the written part of the bar exam; Florida also includes a state-specific multiple-choice component.

Multiple Choice Questions

MBE – The multiple-choice portion of the bar exam tests the following subjects:

UBE/MEE – The MEE can test any of the subjects listed below. Not every subject appears in every exam administration, and some essay questions test more than one subject. For example, one fact pattern could raise issues in both Evidence (such as hearsay) and Criminal Procedure (such as the Confrontation Clause). Conflicts of Law is never tested alone as its own subject but can appear as an issue in questions on Civil Procedure, Family Law, or other subjects.

California – The California bar can test any of the subjects listed here. Professional Responsibility appears every time; otherwise, any subject may show up in any administration. As with the MEE, some essays may test more than one subject.

Other States – Other states test a variety of subjects, including state-specific subjects like state civil procedure. The best way to learn about those bar exams is by consulting the jurisdiction! Many states have extensive materials, including past essay questions and answers, on their state bar websites.

When State Distinctions Matter

Students taking the UBE should answer in accordance with generally-accepted legal principles and do not need to know their state’s distinctions.

Even in California , many of the subjects do not test state-specific law. Specifically: Contracts, Criminal Law, Torts, and Real Property test the same generally-applicable principles of law that you learned during your first year of law school. The law of Trusts is also generally applicable across all states. Similarly, Criminal Procedure and Constitutional Law rely on the U.S. Constitution. This means you do NOT need to know any special California law to do well in any of those subjects! Is addition, Evidence and Civil Procedure problems set in federal court require you to apply the Federal Rules of Evidence and the Federal Rules of Civil Procedure.

However, you will need to know California law for some essays! Answer Community Property, Wills, and Business Associations questions using California law. For Evidence and Civil Procedure problems set in state courts, explain any California distinctions that apply. Similarly, in Professional Responsibility questions, where the ABA Model Rules and the California Rules of Professional Conduct differ, you should demonstrate that you know both rules , and that you recognize any potential difference in application to the facts at hand. Other states may require distinctions, to varying degrees. In ANY jurisdiction, your best bet is to practice from real past bar exam questions to make sure that the work you are doing now is preparing you for the specific bar exam you will be taking down the line!

Read, Set, Go!

Now that you have a clearer idea what the bar exam itself holds in store for you, it’s time to prepare yourself for you bar study. Whether you are on the cusp of graduating from law school or returning to the law after many years away, consider meeting with a BarMax success advisor who can help you craft a plan that’s right for you. Bar prep may feel like a marathon , but you will be ready for the victory that awaits you at the finish line!

#1 Bar Course

Get the #1-Rated Bar review for free.

00 days left to study for the bar..

Get 10% off sent to your inbox.

CRUSH The Bar Exam

How to Tackle Essay Writing on the Bar Exam

How-To-Tackle-Essay-Writing-On-The-Bar-Exam

One skill that is expected to be cultivated and refined during law school is the ability to write well. This makes sense, since good writing will be essential for many legal careers. You will likely need to write memos, client letters, motions, petitions, briefs and other legal documents— so good writing is important! Consequently, the bar exam takes note of this and makes writing an essential component of it. 

Whether you’re taking the Multistate Essay Exam or a state-specific bar exam , you will be writing lots of essays during the bar exam and in your preparation for it. So here’s what you need to know about essay writing on the bar exam and strategies you can implement to improve your score.

Check out the most important bar exam essay writing tips below!

See the Top BAR Review Courses

  • 1. BarMax Review Course ◄◄ Best Overall BAR Review Course + No Discounts
  • 2. Quimbee BAR Prep Course ◄◄ Best Price
  • 3. Kaplan BAR Review Courses ◄◄ Expert Instructors

Multistate Essay Exam (MEE) Jurisdictions

Most states use the Multistate Essay Exam. If you’ll be testing in one of these states, here are the basics you need to know:

There are 6 essay questions in total . This part of the test is 3 hours, so you have 30 minutes per question. Also, the subjects for this portion of the test cover:

  • Partnerships
  • Corporations and limited liability companies
  • Civil procedure
  • Conflict of laws
  • Constitutional law
  • Criminal law and procedure
  • Real property
  • Secured transactions
  • Trusts and future interests
  • Wills and estates

While any of these topics are fair game, these particular topics make up the majority of the MEE:

  • Corporations and LLCs
  • Family law and trusts
  • Future interests

Consequently, you may want to spend extra time preparing for these areas of the law while also studying for the other subjects. 

The good news is that there are guides you can use to determine the most highly tested essay rules. These bar exam study resources will identify these rules and teach you additional rules of law. 

Here’s another important tip: focus your time on these major rules instead of wasting too much energy on nuanced rules that are less likely to be tested. 

Keep reading for more important study tips to help you pass the MEE:

Bar Essays Studying Tips 

The first part of learning how to tackle the essay writing portion of the bar exam is to develop a solid study plan . Your plan should incorporate the following: 

Learn More About the IRAC Method and Format

You may have used a variety of writing styles in law school, such as IRAC, CRAC or CREAC. However, the IRAC structure is the most commonly used one on the bar exam, and is what bar examiners will expect. Hence, you need to be familiar with this writing system:

  • I – Issue
  • R – Rule
  • A – Analysis
  • C – Conclusion

This system ensures that you write concisely and only include the necessary information. It’s not flowery and won’t contain a lot of excess content— which is a good thing, since you’re on such a constrained time limit!

As you practice, read through your answers and label each sentence with an I,R,A, or C . if a sentence cannot be labeled under one of these letters, it probably does not belong.

Practice Essay Writing Each Week

When you spend so much time studying for the bar exam , it may feel tempting to skip practicing the lengthy essay portion of the test. However, this is one of the biggest mistakes made by most test takers. 

Bar essays are an essential component of the test; they can often help leverage a higher score if you don’t do as well on some of the other test portions. Furthermore, while reviewing the rules of law is important, writing about them can show you understand them and know how they apply. 

Basically, don’t leave practicing these essays until the end of your preparation. Instead, make practicing essays part of your weekly study plan!

Bar Exam Essay Practice Tips

Practice Under Timed Conditions

When you first begin practicing the essay portion of the bar exam, you may not want to time yourself so that you can be sure you are spotting all the issues and honing your writing style . However, toward the middle of your study time, you will want to start practicing under timed conditions. 

It is not enough to know how to write a good essay. You need to know how to write a good essay quickly . You need to be able to quickly discuss the most important issues and know when not to elaborate on others.

The best way to study for these questions is to find previous MEE questions and practice them under timed conditions. Then, review the analysis to determine how you did.

Review Rules the Last Two Weeks of Your Study

Focus on memorizing as many rules of law as possible during your last two weeks of studying. You’ll need to be able to recall these basic rules as part of your essay writing without hesitation, so be sure that you can recite rules of law without even thinking about them.

Learn More About The BAR Exam

  • Take These Steps To Pass The Bar Exam!
  • How To Crush The Essay Portion Of The Bar Exam
  • How To Study For The BAR While Working Full Time!
  • How To Pass The BAR After Failing The First Time
  • How To Become A Lawyer

Tips for the Day of the Bar Exam 

Okay, so now it’s the day of the bar exam— you need to know how to truly tackle these questions in the moment of truth. Here’s what you need to do:

Plan The Time You Have for Writing Essays 

Before beginning this portion of the test, you should have a plan on how you will manage your time, such as:

  • First 10 minutes: Read the essay prompt. Maybe read it multiple times. Don’t rush this part; your ability to recall this information will be essential to answering the question. Also, outline your answer as you read through the prompt.
  • Next 15 – 17 minutes: Write your answer.
  • Last 3 to 5 minutes: Review your answer to check for competition and to make necessary edits.

Bar Essay Time Managment

Stick to this timeline for every question. If you start going over 5 minutes on every question, you won’t have enough time to tackle the last question. Ultimately, it’s far better to get out an analysis of all the questions than to answer one question perfectly and not even address another.

Make an Outline

Making an outline can help you organize your thoughts and create a plan on what you will be writing about. Mark up the prompt as you go— you may want to highlight or underline certain information to help your recall later. 

Try to make this outline clear, such as making a bullet list of items related to the prompt. If you run low on time, you can always copy and paste this information to provide a semi-answer to the prompt. Write your rule statement and list the relevant facts that will support your analysis. Also, consider how much time you will need to discuss each subpart of the answer. 

Apply the IRAC Structure

Now it’s time for you to apply what you’ve learned. Use IRAC to fully answer the question. 

How To Use IRAC Method and Format to Crush the Bar Exam Essay Portion

Briefly state the issue in a bolded heading. Issues are usually clearly stated on bar exam essay questions rather than hidden in a fact pattern, so this should be an easy way to pick up points. Restate the issue and move onto the next part of your answer. 

State the rules that apply to the case. This is where rote memorization comes into play, since you need to be able to state the proper rule that applies to the question. Bold key terms to show that you know what rules and terms apply. This will get you the points you need on this section.

The summary of rules should be clear and concise and should demonstrate that you understand what is involved. Only address those rules that actually apply to this case and address the specific question. 

Show how the rule applies, given the particular fact pattern. This will be the longest portion of your answer. However, your analysis should still be shorter than your analysis in your legal writing class. You can pick up (or lose) a lot of points in this portion of the answer! You need to demonstrate that you know how to apply the law to the facts. Generally speaking, the more facts you’re able to explain, the higher your score will be.

Most of the facts in the fact pattern will be there for a reason— and you need to explain why these facts matter in your analysis. Provide a step-by-step analysis of how the facts support your conclusion. You may be able to score extra points by identifying counter-arguments or a majority and minority view. 

Conclusion 

End with a brief conclusion. One sentence is fine here. Perhaps unlike law school exams, there is usually a “right” conclusion. Some writing structures will use a conclusion first and then end with a conclusion, but this is not recommended on the bar exam. If you start with the wrong conclusion, the grader will look for ways to prove why you are wrong while grading your answer; therefore, save your conclusion for the end!

Organize Your Content 

Make your essay simple to read by taking advantage of all the tools at your disposal. Use paragraph breaks to organize your content, creating a clear I, R, A , and C section. Additionally, bold and underline key words and principles of law. Many essay graders will be scanning your work, so make it easy to identify that you understood the legal issues involved by drawing their attention to these key terms.

Also, use transitional words to qualify certain statements and to explain where you are going with your answer. This makes it easier for the grader to follow your analysis, as well as helps you to stay on track.

Answer the Question

Seems obvious, right? Listen:

While it seems simple to just answer the question you are asked, many bar exam essay questions include numerous fact patterns, potential rules of law that apply, and even some red herrings. Be sure that you only answer the question that is asked; don’t go off on a tangent that will not score you any extra points! 

Read over the instructions to the question and follow these instructions, even if that means ignoring something or assuming certain facts are true. Any time you devote to issues that are not relevant to the instructions takes away from time that can score you more points.

Manage Your Time 

Now that you’re in the middle of your answers, keep a close eye on time. It can be tempting to take just a few more minutes to feel you completed a question, but this can come back to haunt you by taking away necessary time from another question. Set alarms if you need to — and are permitted to — so that you know when time is up for each section. Also, you may want to set a reminder a few minutes before your allotted time so that you can quickly wrap up the question before moving on to the next one. 

With that being said, avoid writing a partial essay and then moving onto another one. It can take several minutes to regain your bearings and remember what the essay was about when you switch back and forth. Instead, finish each question in the allotted time and then move onto the next.

Get Discounts On BAR Review Courses!

Kaplan-Best-Bar-Prep-Course-1

Save Up to $700 on Kaplan BAR Review Course

BarMax-Best-Bar-Prep-Course-1-280x280

Enjoy Up to $400 Off on BarMax Course

Quimbee-bar-review-280x150

$300 Savings on Quimbee Bar Review+

Crushendo-Bar-Chart-Logo-280x280-1-280x280

Crushendo Coupon: 10% Off Bar Prep Products

Quick tips for essay writing.

Here’s a quick round-up of tips to keep you on track when preparing your bar exam essays:

  • Read the facts more than once. Don’t rush this part!
  • Don’t write a lengthy, historical background of the law. Instead, make it concise.
  • Don’t write a long analysis regarding policy if the question does not ask for it.
  • Present counter-arguments but spend less time on them than arguments
  • Provide a clear and decisive conclusion.
  • Pace yourself. The two-day bar exam is a marathon, not a sprint. Approach each question with patience and don’t try to rush it.
  • Don’t talk to anyone about your answers. This will undoubtedly make you doubt yourself; you don’t need a hit to your self-confidence at this time!
  • Have a fun plan for what to do after the bar exam to have something to look forward to.

how many essays on the bar exam

So, there you have it— a plan to help you tackle the essay portion of the bar exam. Use these strategies to help boost your score and you will soon be a licensed attorney!

Thanks for reading and good luck on your exam!

Frequently Asked Questions About Bar Essays

How do you write an essay for the bar exam.

There’s a specific structure that bar examiners expect when you write answers to essay questions. This structure is called IRAC, which is short for “Issue, Rule, Analysis, and Conclusion.” When writing a bar essay, try and structure all of your sentences around these four subjects in a way that makes sense.

How many essays are on the bar exam?

The essay portion of the bar exam is called the Multistate Essay Exam, or MEE for short. It is made up of six different essay questions that you must write answers to over the course of three hours. The subjects can vary depending on what test you take, but all are related to the legal field and will require excellent logical reasoning and critical thinking to earn a high score.

How long should bar exam essays be?

Although there may not be a set word limit for your bar exam essay, a good rule of thumb is to write at least 1,000 words for each answer. However, you should avoid padding out your article’s word count with excessively detailed descriptions of legal concepts; stick to the IRAC format and ensure each word in each sentence has a purpose.

Is it better to write or type the bar exam?

There’s no universal answer to this question, since some students will prefer to write by hand and others will prefer typing. However, there are significant benefits to typing your bar exam essay questions over using a pen and paper, such as easy erasing and the ability to copy and paste. However, power issues on rare occasions have forced essay writers to resort to pen and paper, and it makes it impossible to lose progress due to a software error.

COMPARE THE BEST BAR PREP COURSES

how many essays on the bar exam

Valerie Keene is an experienced lawyer and legal writer. Valerie’s litigation successes have included wins for cases involving contract disputes, real property disputes, and consumer issues. She has also assisted countless families with estate planning, guardianship issues, divorce and other family law matters. She provides clients with solid legal advice and representation.

Related Posts

how many essays on the bar exam

Additional Links

Bar Discounts Bar Exam Tips Bar Exam Requirements Policies and Disclosure Terms of Service Contact Us

  • Bar Exam Info
  • Bar Exam Study Materials
  • Legal Services

How-To-Study-For-BAR-While-Working

Banner Image

  • University Library Home
  • Washington and Lee University Research Guides
  • The Library at Washington and Lee University School of Law

Bar Exam Resources Guide

  • Virginia Bar Exam
  • Introduction & First Steps
  • Courses at W&L
  • Books & Study Aids
  • Alabama Bar Exam
  • California Bar Exam
  • Delaware Bar Exam
  • DC Bar Exam
  • Georgia Bar Exam
  • Illinois Bar Exam
  • Maryland Bar Exam
  • New Jersey Bar Exam
  • New York Bar Exam
  • North Carolina Bar Exam
  • Pennsylvania Bar Exam
  • South Carolina Bar Exam
  • Tennessee Bar Exam
  • Texas Bar Exam
  • West Virginia Bar Exam
  • Links to State/Territory Examiners Sites
  • Patent Bar Exam
  • State & Topical Practice Materials

Helpful Information

Contact us!

For additional information on print and electronic resources available to you and the information on this Guide, please contact a librarian at [email protected]

For additional information on W&L bar courses, the bar exam components, studying assistance and tips, creating a bar exam study schedule, etc., please contact Prof. Leila Lawlor at [email protected]

For questions regarding the bar exam application process or materials, please contact Dean Maria Saez Tatman at [email protected]

Looking for a Notary?

In the Law School:

Sue Coffey, Lewis Hall 233A (540-458-8981 or [email protected] )

Brittany Fix, Lewis Hall 348 (540-458-8550 or [email protected] )

Victoria Johnston, Lewis Hall 348 (540-458-8544 or [email protected] )

Brianne Kleinert, Lewis Hall 245 (540-458-8567 or [email protected] )

Emma Martone, Lewis Hall 324 (540-458-8541 or [email protected] )

Sheryl Salm, Lewis Hall 105 (540-458-8482 or [email protected] )

Milea Webb, Lewis Hall 322 (540-458-8584 or [email protected] )

Other Notaries on W&L's campus are listed online . 

Virginia Bar Exam Details

Virginia Bar Exam Schedule:

Tuesday AM: Essays (5 essays, 3 hours)

Tuesday PM: Essays (4 essays + 1 section of short-answer/multiple choice, 3 hours)

Wednesday AM: MBE (100 questions, 3 hours) 

Wednesday PM: MBE (100 questions, 3 hours)

The essay portion of the Virginia bar exam is 60% of your total bar exam score and may test the following subjects:

  • Business Organizations
  • Creditors’ Rights
  • Criminal Law
  • Domestic Relations
  • Federal Practice and Procedure
  • Local Government Law
  • Professional Responsibility
  • Real and Personal Property
  • Uniform Commercial Code
  • Virginia Civil and Criminal Procedure (including appellate practice)
  • Wills, Trusts and Estates

The MBE portion of the Virginia bar exam is 40% of your total bar exam score and tests the following subjects:

  • Civil Procedure
  • Constitutional Law
  • Criminal Law and Procedure
  • Real Property

Study Resources for the Virginia Bar Exam

There are multiple resources that provide sample questions and answers for the Virginia bar exam:

  • Past Virginia bar exams are available to download from the W&L Law Scholarly Commons  and from the VBBE website . 
  • The VBBE does not provide model answers per se but they do provide “ Example Ten-Point Answers ” from past exams on their website. 
  • The William & Mary Law School also compiles suggested answers that have been prepared by a consortium of Virginia law professors. These may be found on the  William & Mary Law School website .
  • << Previous: Texas Bar Exam
  • Next: West Virginia Bar Exam >>
  • Last Updated: Feb 23, 2024 3:45 PM
  • URL: https://libguides.wlu.edu/barexam

NCBE Announces National Mean for February 2024 MBE

MADISON, WISCONSIN, April 2, 2024— The National Conference of Bar Examiners (NCBE) announced today that the national mean scaled score for the February 2024 Multistate Bar Examination (MBE) was 131.8, an increase of more than 0.6 points compared to the February 2023 mean of 131.1. The MBE, one of three sections that make up the bar exam in most US jurisdictions, consists of 200 multiple-choice questions answered over six hours. 

19,496 examinees took the February 2024 MBE, an increase of approximately 1.4% compared to the 19,228 examinees who sat for the exam in February 2023. This increase continues a return toward pre-pandemic examinee numbers that began with last February’s administration.

Approximately 72% of February 2024 examinees were likely repeat test takers and approximately 28% were likely taking the exam for the first time, roughly the same proportion of repeat and first-time test takers as February 2023. [1] All groups of examinees saw performance increases compared to February 2023, with the greatest increase for first-time takers. 

NCBE Director of Assessment and Research Rosemary Reshetar, EdD, commented: “These numbers reflect a continuation of the trend that began last February: we are moving back toward pre-Covid numbers in terms of both the mean and the examinee count. We will likely see an increase in pass rates compared to last February, but we are also still seeing the  effects of the pandemic on examinees who were in law school in 2020, 2021, and 2022.” 

Reliability for the February 2024 exam was 0.93, slightly higher than the reliability for the February 2023 exam and consistent with the 5-year average for February administrations. (Reliability is an indicator of the consistency of a set of examination scores, with a maximum value of 1.0.)

Jurisdictions begin releasing their February 2024 results this week; bar examination pass rates  as reported by jurisdictions are available on the NCBE website. Many jurisdictions are still in the process of grading the written components of the bar exam; once this process is completed, bar exam scores will be calculated and passing decisions reported by those jurisdictions.

More information about the MBE and bar passage rates can be found in the following Bar Examiner  articles:

  • The MBE Mean and Bar Passage Predictions
  • When the Mean Misleads: Understanding Bar Exam Score Distributions
  • Why are February Bar Exam Pass Rates Lower than July Pass Rates?

[1] The first-time and repeat MBE-based test taker information calculated by NCBE is an approximation based on the NCBE Number and biographic data, which has not been used consistently in all jurisdictions across time. Prior to 2022, approximately 10% of examinees could not be tracked with certainty by NCBE as either first-time or repeat takers due to a lack of sufficient biographic information.

About the National Conference of Bar Examiners

The National Conference of Bar Examiners (NCBE), headquartered in Madison, Wisconsin, is a not-for-profit corporation founded in 1931. NCBE promotes fairness, integrity, and best practices in bar admissions for the benefit and protection of the public, in pursuit of its vision of a competent, ethical, and diverse legal profession. Best known for developing bar exam content used by 54 US jurisdictions, NCBE serves admission authorities, courts, the legal education community, and candidates by providing high-quality assessment products, services, and research; character investigations; and informational and educational resources and programs.  In 2026, NCBE will launch the next generation of the bar examination, ensuring that the exam continues to test the knowledge, skills, and abilities required for competent entry-level legal practice in a changing profession.  For more information, visit the NCBE website at  https://www.ncbex.org .

About the Multistate Bar Examination

The Multistate Bar Examination (MBE) is a six-hour, 200-question multiple-choice examination developed by NCBE and administered by user jurisdictions as part of the bar examination, typically given twice each year. The purpose of the MBE is to assess the extent to which an examinee can apply fundamental legal principles and legal reasoning to analyze given fact patterns. The subjects tested on the MBE are Civil Procedure, Constitutional Law, Contracts, Criminal Law and Procedure, Evidence, Real Property, and Torts. In addition to assessing examinee knowledge and skills, the MBE is used to equate the bar exam.  Equating is a statistical procedure used for most large-scale standardized tests to ensure that exam scores retain the same meaning across administrations and over time.  More information about the MBE is available on the NCBE website at  https://www.ncbex.org/exams/mbe/.

About the Uniform Bar Examination

The UBE is a two-day bar examination composed of the Multistate Essay Examination (MEE), two Multistate Performance Test (MPT) tasks, and the Multistate Bar Examination (MBE). It is uniformly administered, graded, and scored and results in a portable score that can be transferred to other UBE jurisdictions. More information about the UBE is available on the NCBE website at  https://www.ncbex.org/exams/ube/ . 41 US jurisdictions currently participate in the UBE, and more than 45,000 examinees took the UBE in 2023.  

  • Jurisdictions
  • Registration
  • ADHD Medical Documentation Guidelines
  • Accommodation Decisions
  • Accommodations FAQs
  • Apply For Test Accommodations
  • Extension Requests
  • How To Prepare Your Request
  • Important Dates for MPRE Test Accomodations
  • Learning Disabilities Medical Documentation Guidelines
  • MPRE Stop-The-Clock Breaks
  • MPRE Test Accommodations Privacy Policy
  • Medical Documentation Guidelines For MPRE Test Accommodations
  • Neurocognitive Disorders
  • Physical and Chronic Health-Related Disabilities
  • Psychological Disabilities
  • Test Conditions
  • Visual Disabilities
  • Test Day Policies
  • Score Portability
  • Minimum Scores
  • Maximum Score Age
  • Local Components
  • UBE Jurisdictions
  • Integrated Question Sets
  • Multiple-Choice
  • Performance Task
  • Content Scope
  • Character & Fitness
  • MBE Score Services
  • MPRE Score Services
  • UBE Score Services
  • Bar Exam Results by Jurisdiction
  • Technical Advisory Panel
  • Covington Award
  • Validity and Fairness Research Award
  • Publications
  • Job Announcements
  • Next Generation of The Bar Exam
  • Diversity and Inclusion
  • News/Resources
  • NextGen Bar Exam
  • Help & Support

Law School Toolbox Podcast Episode 443: Bar Exam Basics for Law Students

In this episode we discuss:

>Structure of the current bar exam and how it's going to change

>Important things to keep in mind for your bar prep

>What are some problem areas we see with people who don't pass the first time?

>How you can set yourself up for success on the exam

>Tools See more + Welcome back to the Law School Toolbox podcast! Today, we're discussing some bar exam basics to help ensure you pass on the first try.

>Tools and resources to use for bar prep

>Bar Exam Toolbox podcast (https://barexamtoolbox.com/)

>Writing of the Week (WOW) Bar Essay Workshop (https://barexamtoolbox.com/writing-of-the-week-wow-bar-essay-workshop/)

>NCBE: National Conference of Bar Examiners (https://www.ncbex.org/)

>AdaptiBar (https://www.adaptibar.com/)

>UWorld (https://www.uworld.com/)

>SmartBarPrep (https://smartbarprep.com/)

>Hidden Potential: The Science of Achieving Greater Things, by Adam Grant (https://adamgrant.net/book/hidden-potential/)

>Podcast Episode 26: Thinking Ahead to Get Ready for the Bar Exam (https://lawschooltoolbox.com/podcast-episode-26-thinking-ahead-get-ready-bar-exam/)

>Podcast Episode 369: Using Spaced Repetition for Your Law School and Bar Exam Studies (w/Gabriel Teninbaum) (https://lawschooltoolbox.com/podcast-episode-369-using-spaced-repetition-for-your-law-school-and-bar-exam-studies-w-gabriel-teninbaum/)

>Podcast Episode 372: The NextGen Bar Exam (w/Doretta McGinnis) (https://lawschooltoolbox.com/podcast-episode-372-the-nextgen-bar-exam-w-doretta-mcginnis/)

>Podcast Episode 413: Preparing for the Bar Exam with Themis and UWorld (https://lawschooltoolbox.com/podcast-episode-413-preparing-for-the-bar-exam-with-themis-and-uworld/)

>Tips for Selecting Core Course Requirements That May Improve Bar Exam Results (https://lawschooltoolbox.com/tips-for-selecting-core-course-requirements-that-may-improve-bar-exam-results/)

Download the Transcript (https://lawschooltoolbox.com/episode-443-bar-exam-basics-for-law-students/)

If you enjoy the podcast, we'd love a nice review and/or rating on Apple Podcasts (https://itunes.apple.com/us/podcast/law-school-toolbox-podcast/id1027603976) or your favorite listening app. And feel free to reach out to us directly. You can always reach us via the contact form on the Law School Toolbox website (http://lawschooltoolbox.com/contact). If you're concerned about the bar exam, check out our sister site, the Bar Exam Toolbox (http://barexamtoolbox.com/). You can also sign up for our weekly podcast newsletter (https://lawschooltoolbox.com/get-law-school-podcast-updates/) to make sure you never miss an episode!

Thanks for listening!

Related Posts

  • Law School Toolbox Podcast Episode 413: Preparing for the Bar Exam with Themis and UWorld   Audio
  • Law School Toolbox Podcast Episode 372: The NextGen Bar Exam (w/Doretta McGinnis)   Audio
  • Law School Toolbox Podcast Episode 369: Using Spaced Repetition for Your Law School and Bar Exam Studies (w/Gabriel Teninbaum)   Audio

Latest Posts

  • Bar Exam Toolbox Podcast Episode 257: Listen and Learn -- Consideration (Contract Law)   Audio 
  • Law School Toolbox Podcast Episode 444: Listen and Learn - Property-Based Intentional Torts   Audio 

See more »

Other MultiMedia by Law School Toolbox

how many essays on the bar exam

Bar Exam Toolbox Podcast Episode 257: Listen and Learn -- Consideration (Contract Law)

Law School Toolbox Podcast Episode 444: Listen and Learn - Property-Based Intentional Torts

Bar Exam Toolbox Podcast Episode 256: Quick Tips -- Budgeting for the Bar Exam

Bar Exam Toolbox Podcast Episode 255: Quick Tips -- Best Bar Exam Study Habits for Takers w/ ADHD

Law School Toolbox Podcast Episode 442: Quick Tips -- How to Get the Most Out of Your Summer Legal Job

Bar Exam Toolbox Podcast Episode 254: Quick Tips -- Best Habits for Studying for the Bar Exam While Working

Law School Toolbox Podcast Episode 441: Exploring the Impact of AI on the Legal Profession (w/Gabe Teninbaum)

Bar Exam Toolbox Podcast Episode 253: Bar Exam Best Practices - Creating Good Habits

Law School Toolbox Podcast Episode 440: Making Good Career Decisions in Law School (w/Sadie Jones)

Bar Exam Toolbox Podcast Episode 252: Listen and Learn -- Property-Based Intentional Torts

Law School Toolbox Podcast Episode 439: Keys to Law School Success with Themis (w/Michele Cooley)

Bar Exam Toolbox Podcast Episode 251: Listen and Learn -- Present and Future Estates (Part 2)

Law School Toolbox Podcast Episode 438: Crafting Less Common Job Application Documents (w/Sadie Jones)

Bar Exam Toolbox Podcast Episode 250: Listen and Learn -- Present and Future Estates (Part 1)

Law School Toolbox Podcast Episode 437: Quick Tips -- Succeeding in Law School with ADHD

Bar Exam Toolbox Podcast Episode 249: Quick Tips -- How to Prepare for the NextGen Bar Exam While Still in Law School

Law School Toolbox Podcast Episode 436: Improving Executive Functioning Skills as a Law Student

Bar Exam Toolbox Podcast Episode 248: Considerations for the NextGen Bar Exam (Plus Questions We Still Have!)

Law School Toolbox Podcast Episode 435: Listen and Learn -- Amendments to Pleadings (Civ Pro)

Refine your interests »

Written by:

Law School Toolbox

Published In:

Law school toolbox on:.

Reporters on Deadline

"My best business intelligence, in one easy email…"

Custom Email Digest

Bar exam data bodes well for February pass rates, after a two-year slump

  • Medium Text

Students attend secondary school exams under COVID-19 restrictions in Berlin

Get a quick look at the days breaking legal news and analysis from The Afternoon Docket newsletter. Sign up here.

Reporting by Karen Sloan

Our Standards: The Thomson Reuters Trust Principles. New Tab , opens new tab

how many essays on the bar exam

Thomson Reuters

Karen Sloan reports on law firms, law schools, and the business of law. Reach her at [email protected]

Read Next / Editor's Picks

Gerber baby food products are seen on shelves at a buybuy Baby store in Libertyview Industrial Plaza, Brooklyn, New York

Industry Insight Chevron

how many essays on the bar exam

Mike Scarcella, David Thomas

how many essays on the bar exam

Karen Sloan

how many essays on the bar exam

Henry Engler

how many essays on the bar exam

Diana Novak Jones

Washington plans to stop requiring bar exam, but alternatives not in place | Fact check

how many essays on the bar exam

The claim: The state of Washington no longer requires passing the bar exam to get a law license

A March 19 Facebook post ( direct link , archive link ) makes a claim about legal education in the Pacific Northwest.

"You no longer need to pass the bar exam to be a lawyer in Washington," the post reads.

A similar version of the post was shared hundreds of times before being deleted.

More from the Fact-Check Team: How we pick and research claims | Email newsletter | Facebook page

Our rating: Partly false

The Washington Supreme Court approved the concept of alternative pathways for law school graduates, law students and law clerks to get law licenses. However, the details of the pathways are still being developed and the timeline to implement those paths is uncertain. Until then, lawyers must pass the bar exam.

New pathways to license

Bar exams , tests to primarily establish that prospective lawyers have a minimum knowledge of the law, have been a gateway to practicing law since the late 18th century in the U.S. However, in recent years advocates have questioned if the exams are good tools for measuring someone's ability to be a competent lawyer.

On March 15, the Washington Supreme Court issued an order that approved "in concept" creating three new pathways to get a law license in that state without passing the bar exam.

However, the court did not spell out a timeline for implementing the new pathways, which are still under development. Jacob Rooksby, dean of the Gonzaga School of Law, told The Spokesman-Review that he did not think it could happen before 2025 at the earliest.

The Washington State Bar Association also says on its website that the court directed it to work on a path to implementation but emphasized the new pathways are not yet in effect.

And the decision doesn't mean, as many social media commenters took it, that anyone can become a lawyer. It still requires extensive training and direct experience in law.

The pathways , developed by task force , emphasize experiential learning. For law school graduates, the alternative pathways would involve an apprenticeship and some additional coursework. Law students could graduate with a license if they complete an internship, earn credits for specific skills and complete a portfolio review. Law clerks will also need to complete an internship and “standardized educational materials and benchmarks to be completed under the guidance of their tutors that dovetail with the requirements of the law school graduate apprenticeship,” according to the announcement of the pathways .

The task force was formed in November 2020 and found that the traditional bar exam “disproportionately and unnecessarily blocks marginalized groups” from becoming lawyers, the statement said . It said the traditional exam is “at best minimally effective for ensuring competent lawyers."

Fact check : No, public hanging is not listed as punishment for treason in US Code

Washington is not the first state to approve alternatives to bar exams. Oregon has begun putting its own universally accessible alternative licensing pathway in place, while Wisconsin and New Hampshire have pathways for graduates of certain law schools. California , Utah , South Dakota , Nevada and Minnesota are among a growing list of states studying or piloting other paths to licensure.

Our fact-check sources:

  • The Washington State Bar Association, accessed March 29, Latest News
  • Spokesman-Review, March 15, Supreme Court: Bar exam will no longer be required to become attorney in Washington State
  • Washington Supreme Court, March 15, Order 25700B711
  • Washington State Bar Licensure Task Force, Feb. 28, A Proposal for the Future of WA State Bar Admissions Updated Following Public Comment  
  • Washington State Courts, March 15, Supreme Court Approves Alternative Pathways to Lawyer Licensure in Washington State

Thank you for supporting our journalism. You can subscribe to our print edition, ad-free app or e-newspaper here .

USA TODAY is a verified signatory of the International Fact-Checking Network, which requires a demonstrated commitment to nonpartisanship, fairness and transparency. Our fact-check work is supported in part by a grant from Meta .

how many essays on the bar exam

More than 12,000 register for 2024 Bar Exams

MANILA, Philippines – The Supreme Court revealed on Monday that 12,246 law graduates have enrolled for the 2024 Bar Examination.

The deadline for the application to take the 2024 Bar Examination was last April 5.

Applications are currently being processed, which is why it is still not known how many first-time takers, repeaters, or those who will take the exam for the second time, and refreshers or those who took the Bar exam three times or more.

The digitalized Bar examination is scheduled on September 8, 11, and 15 in multiple locations nationwide.

Similar to the 2023 Bar examination, there will be six core subjects divided on the three day exam.

On September 8, exam will be on Political and Public International Law and Commercial and Taxation Laws; Civil Law and Labor Law, and Social Legislations on September 11 and Criminal Law and Remedial law, Legal and Judicial Ethics with Practical Exercises on September 15.

Applicants need to regularlycheck their Bar Application Registration System and Tech Assistance (BARISTA) account for updates on their application and examination venue.

The Office of the Bar Confidant (OBC) will also email the registrants regarding issues with their documents or whether they already need to send the Supreme Court physical copies of their uploaded documents.

Matters regarding the guidelines and rules of conduct for the 2024 Bar Examinations will be announced in subsequent Bar bulletins.

In the 2023 Bar exams, 3,812 out of 10,387 examinees passed, registering a passing rate of 33.77 percent.

Paulo Batulan, a graduate of the University of San Jose-Recoletos (USJ-R), landed in the top 10 of the 2023 Bar Exams and was the only Cebuano to achieve this feat.

RELATED STORIES

2023 Bar exam produces 3,812 new lawyers in PH

Bar Exams 2023: Cebu university graduate is No. 8 among 3,812 passers

bar exams

IMAGES

  1. Four Tips on How To Write A Great Bar Exam Essay

    how many essays on the bar exam

  2. Grading Bar Exam Essays

    how many essays on the bar exam

  3. The ULTIMATE Bar Exam Essay Guide

    how many essays on the bar exam

  4. How to write better bar exam essays, faster

    how many essays on the bar exam

  5. HOW TO ANSWER BAR EXAM ESSAY QUESTION IMPRESSIVELY.docx

    how many essays on the bar exam

  6. How to Practice Essays in the Final Weeks Before the Bar Exam

    how many essays on the bar exam

VIDEO

  1. CA Bar Exam Essay Workshop Series: Property

  2. Free BarMD California Essay Workshop: Contracts

  3. Essay Writing for the Bar Exam by Hugh Reed

  4. DO THESE THINGS 1 WEEK BEFORE THE BAR EXAM || MY TOP 5 TIPS

  5. How to write a high scoring Essay for the California Bar Exam

  6. CA Bar Exam Essay Workshop with Criminal Law and Procedure Essay insight with BarMD

COMMENTS

  1. Exactly How Many Essays Are On The Bar Exam?

    When wondering how many essays are on the bar exam, make sure you find the answer to this question for your specific bar exam. For those in Uniform Bar Exam jurisdictions: You will be taking the MEE as your essay component. In these states, there are six essays on the bar exam. Each state administers the same six essays.

  2. 13 Best Practices for Grading Essays and Performance Tests

    This article originally appeared in The Bar Examiner print edition, Winter 2019-2020 (Vol. 88, No. 4), pp 8-14. By Sonja Olson. When grading essays and performance tests for the bar examination, fairness, consistency, and focus are the cornerstones of good grading.

  3. Prioritizing Rules for the Bar Exam (and How to Dominate Essays)

    1) Don't write like a lawyer. Write like a bar taker. 2) Organize the issues. Prioritize memorizing the rules and issues in this order…. 1) Issues and rules that have appeared in past exams, including main issues, sub-issues, defenses. 2) Issues and rules deemed important.

  4. MEE Bar Exam

    The Multistate Essay Examination (MEE) consists of six 30-minute questions. Developed by NCBE, the MEE is administered by user jurisdictions as part of the bar examination on the Tuesday before the last Wednesday in February and July of each year. The MEE is only one of a number of measures that a board of bar examiners may use in determining ...

  5. Common Bar Essay Problems and How to Avoid Them

    The answers that score the highest have some telltale traits that really start standing out the more of them you read. We here at the Bar Exam Toolbox have seen a lot of bar essays: top scores, failing scores, and everything in between. We've also come up with some common pitfalls that tend to make an essay score lower.

  6. Bar Exams

    The most common testing configuration consists of a two-day bar examination, one day of which is devoted to the Multistate Bar Examination (MBE), a standardized 200-item test covering six areas (Constitutional Law, Contracts, Criminal Law, Evidence, Real Property, and Torts). The second day of testing is typically comprised of locally crafted essays from a broader range of subject matters.

  7. How You Can Master Essays on the Bar Exam

    The 3 Components of the Bar Exam. The bar exam consists of three main components in most jurisdictions: multiple choice, essays, and a performance test. Excelling on each requires different approaches. The multiple choice section (called the "Multistate Bar Exam") is the toughest and counts for a significant part of your total score. The ...

  8. What Is On the Bar Exam?

    The Multistate Bar Exam (MBE) questions consist of a brief fact pattern, a question, and four multiple-choice answers. Test takers must answer 200 MBE questions over a span of six hours. The test is broken up into two sections, a morning and afternoon session with 100 questions each. Only 175 MBE questions are scored.

  9. How to Practice Essays in the Final Weeks Before the Bar Exam

    After every timed practice essay, set aside at least 30 minutes to review your answer and compare it to sample answers from past bar exams. In most states, samples released by the state bar typically go far beyond what is needed to achieve a passing score. Nevertheless, you'll want to check your practice essay against these answers and make ...

  10. Demystifying the Bar: What Exactly Is On the Bar Exam?

    Both the MEE and MPT are drafted by the National Conference of Bar Examiners (NCBE) and are identical in all UBE states. California: The first day of the California Bar Exam consists of 5 essays questions of one hour each, followed by one California Performance Test. There is a break for lunch between the third and fourth essay questions.

  11. How to Tackle Essay Writing on the Bar Exam

    Next 15 - 17 minutes: Write your answer. Last 3 to 5 minutes: Review your answer to check for competition and to make necessary edits. Stick to this timeline for every question. If you start going over 5 minutes on every question, you won't have enough time to tackle the last question.

  12. Scope of the California Bar Examination

    The MBE is developed and scored by the National Conference of Bar Examiners (NCBE). This portion of the General Bar Examination is an objective six-hour examination containing 200 questions, divided into two, three-hour sessions during each of which 100 questions are administered. The MBE tests seven subjects: Civil Procedure, Constitutional ...

  13. Bar Exam Resources Guide

    Virginia Bar Exam Schedule: Tuesday AM: Essays (5 essays, 3 hours) Tuesday PM: Essays (4 essays + 1 section of short-answer/multiple choice, 3 hours) Wednesday AM: MBE (100 questions, 3 hours) Wednesday PM: MBE (100 questions, 3 hours) The essay portion of the Virginia bar exam is 60% of your total bar exam score and may test the following ...

  14. PDF July 2021 Bar Exam FAQs

    • The first day of the General Bar Exam will be comprised of five one-hour essay questions and a 90-minute Performance Test question, with a one-hour lunch break. For the morning session, you may log into the system as early as 8:00 a.m., but no later than 8:30 a.m. For the afternoon session on day 1, you may log

  15. NCBE Announces National Mean for February 2024 MBE

    The National Conference of Bar Examiners (NCBE) announced today that the national mean scaled score for the February 2024 Multistate Bar Examination (MBE) was 131.8, an increase of more than 0.6 points compared to the February 2023 mean of 131.1. The MBE, one of three sections that make up the bar exam in most US jurisdictions, consists of 200 multiple-choice questions answered over six hours.

  16. About the Ohio Bar Exam

    The Ohio Bar Exam is held every February and July. It is a three day test that begins on Tuesday morning and ends Thursday at noon. It is composed of three different parts: Twelve essay questions. Multistate Bar Exam, 200 multiple choice questions, and. Multistate Performance Test, two closed-universe writing problems.

  17. Law School Toolbox Podcast Episode 443: Bar Exam Basics for Law

    Other MultiMedia by Law School Toolbox. Bar Exam Toolbox Podcast Episode 256: Quick Tips -- Budgeting for the Bar Exam . Bar Exam Toolbox Podcast Episode 255: Quick Tips -- Best Bar Exam Study ...

  18. PDF California Bar Examination

    ESSAY QUESTIONS 1, 2 AND 3 California Bar Examination Answer all 3 questions; each question is designed to be answered in one (1) hour. Your answer should demonstrate your ability to analyze the facts in the question, to tell the difference between material facts and immaterial facts, and to discern the points

  19. Bar exam data bodes well for February pass rates, after a two-year

    The national average score on the Multistate Bar Exam — the 200 multiple-choice question portion of the exam — ticked up 0.7 of a point to 131.8, according to the National Conference of Bar ...

  20. Washington to create alternative to state bar exam

    The claim: The state of Washington no longer requires passing the bar exam to get a law license. A March 19 Facebook post (direct link, archive link) makes a claim about legal education in the ...

  21. Over 12,000 register for 2024 Bar Exams

    Matters regarding the guidelines and rules of conduct for the 2024 Bar Examinations will be announced in subsequent Bar bulletins. In the 2023 Bar exams, 3,812 out of 10,387 examinees passed ...

  22. More than 12,000 register for 2024 Bar Exams

    Matters regarding the guidelines and rules of conduct for the 2024 Bar Examinations will be announced in subsequent Bar bulletins. In the 2023 Bar exams, 3,812 out of 10,387 examinees passed ...